What is the remainder R?

What Is The Remainder R?

Answers

Answer 1

What is remainder R when the polynomial p(x) is divided by (2x - 1)? is (2x - 1) factor of p(x)?

p(x) = 2x² + 3x + 1

Answer :

R = 3

and (2x - 1) not factor of p(x) because, if (2x - 1) factor of p(x) remainder R when the polynomial p(x) divided by (2x - 1) is R = 0

so the answer is option A

Step by step explanation :

« Attached »

What Is The Remainder R?

Related Questions

Tres rollos de tela de 30,48 y 72 metros de largo se requieren cortar para hacer pedazos iguales y de mayor longitud¿cual será el largo de cada pedazo?

Answers

Answer:

Desde los 30 metros: 5 piezas de 6 metros

Desde los 48 metros: 8 piezas de 6 metros

Desde los 72 metros: 12 piezas de 6 metros

Step-by-step explanation:

Cada rollo es divisible por 6

what is 8f to the 4th power/2f to the third power in parentheses squared

Answers

The parenthesis squared mean it is the 4th power to the 8th

help me please!!!!!!!!!!!!!!!!!!!!!!!!!!!!

Answers

Answer:77

Step-by-step explanation:

I need help with this asap. thank u

Answers

Answer:

1/3

Step-by-step explanation:

1. When we divide any x-input by the y-input, we get 3. This is the same as multiplying the x-input by 1/3 to get the y-input. Let's see if it works!

[tex]3 * \frac{1}{3} = \frac{3}{3} = 1[/tex] [tex]6 * \frac{1}{3} = \frac{6}{3} = 2[/tex] [tex]12 * \frac{1}{3}= \frac{12}{3}=4[/tex] [tex]15*\frac{1}{3}=\frac{15}{3} =5[/tex]

2. Great! All our x-inputs and y-inputs match up, meaning the constant of proportionality, k, is 1/3.

Find the value of x

solving steps

Answers

Answer:

x = 8

Step-by-step explanation:

Supplementary angles add up to 180 degrees to make a straight line.

The red square in one of them indicates that it is 90 degrees on their side which means that it is 90 degrees on the other where (6x+2) and 40 degrees are.

(6x+2) + 40 = 90

6x + 42 = 90

6x = 48

x = 8

Write this ratio as a fraction in lowest terms.
70 minutes to 60 minutes

Answers

ANSWER:

10 MINUTES

EXPLANATION:

BECAUSE IF YOU SUBTRACT 70 AND 60 THE ANSWER WILL BE 10.

7060=10

HOPE IT HELPS YOU

70 min to 60 min. divide both by 10. the lowest terms you can get for this answer would be 7 min to 6 min. i hope this helps !

Katie bought 4 sweaters that each cost the same amount and 1 skirt that cost $20. The items she bought cost a total of $160 before tax was added. What was the cost of each sweat?

Answers

Answer:

$35

Step-by-step explanation:

first we need to find the amount of money the sweaters cost alone.

so we first subtract the total (160) - the skirt price (20) which is 140

now we divide it by 4 to find the price of each seater, as they all cost the same.

so 140 divided by 4 = 35

answer: $35

one number is 15 more than a second number.The sum of the number 115. What are the numbers?

Answers

The first number is 65
The second number is 50

X= first number
Y= second number

X+Y= 115 > or 2y+15= 115 > subtract 15 from both side

2y= 100> divide by 2 on each side> y=50

There you have solved for y, now solve x

X= 50+15 > which is 65

So therefore, the first number is 65 and the second is 50.

what times 3 equals 0.5?

[tex](x)3=0.5[/tex]

Answers

Answer:

.16666667

Step-by-step explanation:

(x)3=0.5

divide both sides by 3

x    =.16666667

Jeff wants to buy tickets for a football game for his friends. The stadium is selling tickets for $7 each. The stadium requires a
person to buy at least 4 tickets, and at most 8 tickets. Let x represent the number of tickets, and y represent the total cost.
What is a reasonable range for the price Jeff would pay for tickets?
Group of answer choices



{11, 18, 25, 32, 39}

{28, 35, 42, 49, 56}

{4, 5, 6, 7, 8}

{28, 42, 56, 70, 84}

Answers

Your answer is D){28, 42, 56, 70, 84}

To answer this question you just have to make sure the amount he pays is a multiple of 7.If you know you times table then this should be easy

Times table of 7:

7,14,21,28,35,42,49,55,63,70,77,84.

Now you just need to find an answer choice who has these numbers.Lets check it out!

A) 11 isn't a multiple of 7

B) 56 isn't a multiple of 7

C) None of these are multiples of 7 except 7 so this is also incorrect

D) This is correct because they are all multiples of 7

Hope this makes sense.

At the garage sale, each book is priced the same. You purchased 3 fiction books, 5 picture books, 2 non fiction books.and 4 puzzel books. write an expression in simplest form that represents the total amount of money the order will cost

Answers

14x = the total cost of the order


explanation:
since every book is priced the same, you can add them together to get 14. x = the cost of one book, but since we don’t know the cost of one book, we use the variable x. so, 14x is equal to the total cost of the order. hope this helped!

(-15x + 21y) divide by 3

Answers

Hey! Your answer to this would be -5x + 7y

please help will give brainliest immediately

Answers

Answer:

5

Step-by-step explanation:

H                       15                          J

------------------------------------------------

         5           I                10

15-10 =5

i need answer please​

Answers

Answer:

2nd one

Step-by-step explanation:

(x,y)

- 3 off of y (because it's going down)

-5 off of x (because it's going left)

Answer:

A=(-4,-2)

B=(-1,-1)

C=(-2,2)

Step-by-step explanation:

A = (1,1) = (-5+1) and (-3+1) = (-4,-2)

B = (4,2) = (-5+4) and (-3,+2) = (-1,-1)

C= (3,5) = (-5+3) and (-3+5) = (-2,2)

what is the probability of selecting four red cards from a standard deck if each card is replaced before the next one is selected

Answers

Answer:

1/16

Step-by-step explanation:

1/2 * 1/2 * 1/2 * 1/2

=1/16

Can someone help me out with this please?

Answers

Answer:

[tex]3781/9000[/tex]

Step-by-step explanation:

[tex]0.42\overline{1}=42/100+0.00\overline{1}[/tex].

Recall that [tex]0.\overline{1}=1/9[/tex].

So, we want [tex]42/100+1/9*1/1000[/tex].

This simplifies to [tex]21/50+1/9000[/tex].

We know that [tex]21/50=(21*180)/(50*180)=3780/9000[/tex].

So, our expression is [tex]3780/9000+1/9000=3781/9000[/tex], which can't be simplified more.

So, our answer is [tex]\boxed{3781/9000}[/tex] and we're done!

help
Which lines are perpendicular to the line y –1=1/3(x+2)?

check all that apply

y + 2 = –3(x – 4)

y − 5 = 3(x + 11)

y = -3x – Five-thirds

y = One-thirdx – 2

3x + y = 7

Answers

Perpendicular lines have negative reciprocal slopes (in which the product of those two slopes = -1). The negative reciprocal of 1/3 is -3.

Therefore, the following choices are perpendicular to y — 1 = 1/3(x + 2):


You’ll have to transform option 1 into its slope-intercept form to see whether its slope is a negative reciprocal to 1/3:

OPTION 1:

y + 2 = -3(x - 4)
Distribute -3 into the parenthesis, and subtract 2 from both sides:
y + 2 - 2 = -3x + 12 - 2
y = -3x + 10

OPTION 5:
You’ll also have to transform option 5 into its slope-intercept form:

3x + y = 7
Subtract 3x from both sides to isolate y:
3x - 3x + y = - 3x + 7
y = -3x + 7


OPTION 3:
Option 3 also has a negative reciprocal slope of -3:

y = -3x - 5/3

Therefore, the following options are the correct answers:

Option 1: y + 2 = -3(x —4)
Option 3: y = -3x - Five-thirds or y = -3x - 5/3
Option 5: 3x + y = 7


Please mark my answers as the Brainliest if you find my explanations helpful:

the mean of eight numners is 41 the mean of two of the numbers is 29 work out the mean of the other six numbers

Answers

Answer: 270/6 =45

Step-by-step explanation:

There are 3 different sets of numbers involved here.

A set of six, a set of two and the set of all eight.

Each set has its own mean.

Plz helppppp me I need to know how to do this for a test

Answers

The answer you’re looking for would be D. (8,4).

Good luck on the test.

please help me or I'm gonna fail math​

Answers

Answer:

As follows,

Step-by-step explanation:

a) Completely wrong,

Correction

x^2-7x+12

x^2-4x-3x+12

x(x-4)-3(x-4)

(x-3)(x-4)

b)Partially wrong

Correction

35x^2-5x

5x(7x-1)

Based on the data set shown, which of the following is a true statement?
-1,-1,0, 1, 1, 1, 1, 2, 2, 2, 3

Answers

Answer:

3

Step-by-step explanation:

Answer:

Agree with the person on top

Step-by-step explanation:

Mean < median

help please and thank you

Answers

Answer:

12

Step-by-step explanation:

25.2 / 2.1

The divisor should be a whole number. Multiply the dividend and the divisor by 10.

= 252 / 21

= 12

how many digits do you need to write all of the 2-digit numbers?

Answers

Answer:

I think it's 180 digits

Answer:

2

Step-by-step explanation:

any 2 digits using 0 through 9

I will brainless who can answer this all pleaseeeeej​

Answers

27) A

28) C

29) B

30) T

Factorise the link expression 5(x- 2) - y(2-x)

Answers

Step-by-step explanation:

5(x- 2) - y(2-x)

5(x- 2) - -y(x-2)

5(x- 2) + y(x-2)

=(5+y) (x-2)

Can someone please help me ASAP NO LINKS!!!

Answers

Answer:

-7 3/4 (or it can be written as -31/4, an improper fraction)

Step-by-step explanation:

[tex]\left(\frac{1}{2}\right)^2-6\left(2-\frac{2}{3}\right)[/tex] [tex]\frac{1}{4} - 6(2-\frac{2}{3})[/tex] [tex]\frac{1}{4}-6(1\frac{1}{3})[/tex] [tex]\frac{1}{4} - 6 * \frac{4}{3}[/tex] [tex]\frac{1}{4} - \frac{24}{3}[/tex] [tex]\frac{1}{4} - 8[/tex] [tex]\frac{-31}{4}[/tex] [tex]-7\frac{3}{4}[/tex]

Therefore, the answer is -7 3/4 as a mixed number and -31/4 as an improper fraction (they're the same thing).

You Use the distributive property to find the product 3/4 x 2 1/3​

Answers

Answer:

1 1/4

Step-by-step explanation:

3/4 times 2 1/3 you make that an omproper fractions

Consider 2 3/4 divided by 1/8

(A) write a real world problem for the division

(B) create and explain a model for the division

(C) find the quotient show your work or explain your reasoning

Answers

Answer:

Part A. Mary has 2 3/4 cups of sugar to make cookies. The recipe requires 1/8 cup of sugar for one portion of cookies. How many portions of cookies can Mary make?

Part B.

 portions.

Part C. The quotient is 22 and this number represents the amount of portions of cookies that Mary can make.

Step-by-step explanation:

tell whether the following sequence is geometric or not. If geometric find r

hi can someone please answer my question i really need the complete step-by-step solution. thank you. ​

Answers

Answer:

not geometric because of the 4 at the beginning

Answer:

No

Step-by-step explanation:

We can observe the difference of terms is constant:

d = 1/3

The sequence is arithmetic, not geometric.

harry and his 3 friends decided to order pizza, and the cost was $42.24. How much money will each have to pay?

Answers

Answer:

$10.54

Step-by-step explanation:

42.24 divided by 4

Other Questions
Before Sarah does open her account, she sees her grandmother for the holidays. Grandma wants to contribute to Sarahs ROTH and even matches her contribution, giving her an additional $500 to invest. Now that Sarah has an initial investment of $1000, how much would Sarah have at age 68 based on her predicted rate of return from historical data What is an equation of the line that passes through the point (-6,-1) and is perpendicular to the line 6x-y=7 which was allowed under the new system of government in Japan following world war 2 SPANISH CROSSWORD: I really need some help getting this Capitulo 1A1A- 8 crossword done! Will give points, thanks, and status! __ she __ any sisters?A. Does/hasB. Does/haveC. Has/haveD. Is/having Solve for x: 15x=x+2 Tom tells Greta that his father, an astronomer, is planning a scientific experiment to study another galaxy. Using a powerful telescope, Tom's father will record data of shifts in light patterns and track their different sources every 30 seconds. He will later study the data. Greta says that this is not an experiment. Which of these reasons supports Greta's statement?It is not performed in a lab.There are too many variables.It does not have a control group.It uses only theoretical predictions. Which of the following statement is true about recombinant DNA Which is the outermost skin of the earth?: 4[3(5+2)]=95-4x3 is this statement true or false Question 4 (2 points)A pamphlet is considered to be one of the earliest forms ofO religious literatureO protest literatureAmerica's first dramatic literaturean Appeal to Authority/A Credible Source (Ethos) what is 7 1/2 divided by 5 5/8 WILL GIVE BRAINLIESTHow did inflation in rome affect the citizens trying to pay taxes HELPPPP! The Senate has 100 members, ______ from every stateWhat goes in the empty slot? What was responsible for both the rise of towns and the rise of the merchant class? * is mixing acid and base a chemical change or a physical change Does anyone know this? If Mac were to get 76 apples from her friends and then get 3 more from the store, and then find two absolutely fine apples in the sewers, then go for a walk and find 7500 apples in someone's car, how many healthy apples would she find? Bet you can't solve it ;) A store owner sold 6 sodas for $12.00 and 8 bags of chips for $6.00. What is the difference between theunit price of a paint brush and the unit price of a pen? i need an economy for virginia?